X=2 y=5
2x^0y^-2
Please help

Answers

Answer 1

Answer:

2/25

Step-by-step explanation:


Related Questions

Natalie found a tennis ball outside a tennis court. She picked up the ball and threw it over the 12-foot fence into the court. The height of the ball, h, and time t seconds is given by the equations h(t) = -16t^2 + 18t +5. Will she make it iver the fence?

Answers

Answer:

Natalie cannot make it over the fence.

Step-by-step explanation:

After reading the statement, we must demostrate if the following inequation is possible, that is:

[tex]-16\cdot t^{2}+18\cdot t+5 \ge 12[/tex]

After some algebraic handling we find this equivalent expression:

[tex]16\cdot t^{2}-18\cdot t +7\le 0[/tex] (1)

The polynomial is factorized by the Quadratic Formula:

[tex]\left(t-0.563-i\,0.348)\cdot (t-0.563+i\,0.348)\le 0[/tex]

Since all roots are complex, we conclude that Natalie cannot make it over the fence.

School guidelines require that there must be at least 2 chaperones for every 25 students going on a school trip. How many chaperones must there be for 82 students? A. 6 chaperones B. 41 chaperones C. 7chaperones D 3 chaperones

Answers

Answer:

A. 6 chaperones

Step-by-step explanation:

Since it's 2 chaperones per 25 students you would need to see how many times 25 can fit into 82

82/25 = 3.28

You can round that number to 3

Then you'll multiply 3 by 2 to get your number of chaperones for the school trip

3 x 2 = 6

There's still students left but not enough for there to be another chaperone so the answer is 6

MARKING AS BRAINLIST PLEASE HELP ONLY ANSWER IF YOU KNOW

Answers

The answer is “B” this is the correct answer because in asa we need to identify another angle which would be angle d and p

How many 1 fourths are in 3 fourths?

Answers

Answer:

12

Step-by-step explanation:

This question means 3/(1/4). The rule for division of fractions is invert the denominator and multiply. 3/(1/4) = 3 * 4 = 12. Another way to think of it is to imagine you have 3 pies each divided into quarters.

Guess my number between 1-5 and u will get brainliest

Answers

Answer:

3

Step-by-step explanation:

Answer:

1

Step-by-step explanation:

Please please please someone help me

Answers

Answer:

15

Step-by-step explanation:

Answer:

x=15

Step-by-step explanation:

Substutting in q(x) makes 3=4/5x-9

Add 9 to both sides to get 12=4/5x

Divide both sides by 4/5 and get 15=x

Thus, x=15

Which equation is y = (x + 3)2 + (x + 4)2 rewritten in vertex form

Answers

Answer:

y=2(x+3.5)^2+0.50

Step-by-step explanation:

we have

y=(x+3)^2+(x+4)^2

y=x^2+6x+9+x^2+8x+16

y=2x+14x+25

rewrite now in vertex form

Group terms that contain the same variable, and move the constant to the opposite side of the equation

y-25=2x^2+14x

Factor the leading coefficient

y-25=2(x^2+7x)

Complete the square. Remember to balance the equation by adding the same constants to each side

y-25+24.50=2(x^2+7x+3.5^2)

y-0.50-2(x^2+7x+12.25)

Rewrite as perfect squares

y-0.50=2(x+3.5)^2

y=2(x+3.5)^2+0.50

the vertex is the point (-3.5,0.50)

therefore the answer is

y=2(x+3.5)^2+0.50

Pleas help me on this

Answers

Answer:

3x

Step-by-step explanation:

x+x+x

=3x

if i am wrong than sorry

I think 3x I’m not completely sure tho!

round the number 1 560 to the nearest thousands place​

Answers

Answer:

2000

Step-by-step explanation:

Answer:

2,000

Step-by-step explanation:

the reason of this is the one becomes a two so the other numbers go to zero

then there is a possible another answer 1060 but i think it is wrong so the answer is 2,000.

if you like my question please send brainiest thanks.

I WILL GIVE BRAINILEST AND 5 STAFS IF CORRECT

Use the expression 9(7 + 2x) to answer the following:

Part A: Describe the two factors in this expression. (4 points)

Part B: How many terms are in each factor of this expression? (4 points)

Part C: What is the coefficient of the variable term? (2 points)

Answers

Answer:

Part A: Describe the two factors in this expression. (4 points) The factors are (1) the constant coefficient 9 and (2) the binomial (7+2x).

Part B: How many terms are in each factor of this expression? (4 points) The first factor (multiplicand), 9, has one term. The second factor (multiplicand), (7+2x), has two terms (and is thus called a binomial).

Part C: What is the coefficient of the variable term? (2 points) The only such coefficient is 2.

What is the answer to: 5/2 - 4/3

Answers

Answer:

7/6

Step-by-step explanation:

5/2-4/3

=(5*3-4*2)/2*3

=(15-8)/6

=7/6

At a store, socks are sold with a different number of socks in each package. Which package has the lowest price per pair of socks?

A.
4 pairs for $4.99

B.
6 pairs for $7.99

C.
8 pairs for $10.99

D.
12 pairs for $15.99

Answers

B hope this helps divide each of the answers choices by the total by number of pairs

Answer:

I'm sure the correct answer is A. 4 pairs for $4.99.

Step-by-step explanation:

Please tell me if I'm wrong. And also the answer is not B for other's that may think it is. I hope you find this helpful, have a great day.

2x100x3x0 solve lol :)

Answers

Answer:

the answer is 0

Step-by-step explanation:

2x100x3x0 equals and levels out to 0 :)

Answer:

0

Step-by-step explanation:

100 x 2 = 200

200 x 3 = 600

600 x 0 = 0

What is an equation of the line that passes through the points (0, -4) and
(4.6)? Put your answer in fully reduced form.

Answers

Answer:

5-8. 10- 8 39-66 87 9-99 01 45,

Percy's Pizzeria made 4 pizzas with pepperoni and 59 pizzas without pepperoni. What is the ratio of the number of pizzas with pepperoni to the number of pizzas without pepperoni? I had got 15/59 i wanna know if it is correct.

Answers

Answer:

4 : 59

Step-by-step explanation:

Close. But good try

So what is "ratio?"

Well seems like you got 15/59 i think you meant 15:59 right? ratio has a : in it.

But, you see there is only 4 pizzas with pepperoni in it so actually it should be

4 : 59

Good attempt anyways Trying is always better than nothing.

His problem probably won’t be too hard! Help please I need it ASAP ​

Answers

Answer:

24

Step-by-step explanation:

24 is the answer ksnskskskndndkdnd

The answer is 24! Hope this helped

If f(x) = 4x and g(x) = x^3, then what is f(g(x))?
a. 4x+x^3
b. 4x^4
c. 4x^3
d. (4x)^3

Answers

Answer:

f(g(x)) = 4x³

Hence, option C is true.

Step-by-step explanation:

Given

[tex]f(x) = 4x[/tex][tex]g\left(x\right)\:=\:x^3[/tex]

Determine:

f(g(x)) = ?

solving the f(g(x))

f(g(x)) = f(x³)

         = 4x³

Therefore,

f(g(x)) = 4x³

Hence, option C is true.

how do you simple this​

Answers

Answer:

Turn the first fraction into a mixed number then simply

Step-by-step explanation:

Turn into a mixed fraction my mutplyung the 3 and then adding the one and then go from there I’m pretty sure

Given the points P(2, –6) and R(8, 3), what is the component form of Vector P R?

⟨6, -3⟩
⟨10, -3⟩
⟨6, 9⟩
⟨10, 9⟩

Answers

Answer:

C

Step-by-step explanation:

The component form of vector PR is ⟨6, 9⟩.

How to determine the component form of vector PR

To find the component form of vector PR, we need to subtract the coordinates of point P from the coordinates of point R.

The x-component of the vector is obtained by subtracting the x-coordinate of point P from the x-coordinate of point R:

x-component = 8 - 2 = 6

The y-component of the vector is obtained by subtracting the y-coordinate of point P from the y-coordinate of point R:

y-component = 3 - (-6) = 3 + 6 = 9

Therefore, the component form of vector PR is ⟨6, 9⟩.

Learn more about vector at https://brainly.com/question/25705666

#SPJ2

Kyle’s school needs to buy posters for a fundraiser. The school has a budget of $147. Each poster costs $13. About how many posters can his school buy? Solve this problem any way you choose

Answers

Answer:

Eleven. or 11.

Step-by-step explanation: 147 divided by 13 is 11 with a remainder of 4. He would have 4 dollars left over. If he bought 11 posters.

What is the value of x in this figure?

Answers

Answer:

65

Step-by-step:

You would do 180-48-67= 65

How many yards are there in 14 1/2 miles?

Answers

Answer:

25520

Step-by-step explanation:

I need help with this to

Answers

Answer C is correct.

Explanation:

Let X = -2 and Y = 10

Y = -3x + 4
10 = -3*(-2) + 4
10 = 10

Which checks out.
C is correct hope this helps <3

The expression 2y+ 2.6 is equal to ? for y=5

Answers

Answer:

Its 12.6

Step-by-step explanation:

first we multiply 2 by 5 and get 10. Then we add 10 to 2.6 and get 12.6 for your answer.

Answer: 12.6

Step-by-step explanation:

2(5)+2.6

10+2.6

12.6

The system of equations is graphed on the
coordinate plane.
y = x - 1

y = -2x - 4

Enter the coordinates of the solution to the
system of equations in the boxes.
(___,___)

Answers

-1,-2 because they intersect

The number line shows the locations of points A and C.
If point B is 1/3 of the way from point A to point C, what is the coordinate of point B ?

Answers

Answer:

Point B would be at 2.5

Step-by-step explanation:

The points have an 8 umber distance so numbers 2 and 3 are the middle numbers and the middle of those numbers would be 2.5.

Answer:

2.5

Step-by-step explanation: i did it on a test and got it right

. The ratio of boys to girls in Ms. Cunningham's class is 2 to 3. There are 21 girls in the class. What is the total number of students in Ms. Cunningham's class?

Answers

Answer:

14

Step-by-step explanation:

21 times 2 = 42

42 divided by 3 = 14

Which equation matches the graph y=-1x+3 , y=1x, y1x+3, y=-1x can you guys tell me which equation matches the graph with explanation

Answers

Answer:

y = 1.x + 3

Step-by-step explanation:

Let the equation of the line is,

y = mx + b

Here, m = slope of the line

b = y-intercept

Slope of a line passing through two points [tex](x_1,y_1)[/tex] and [tex](x_2,y_2)[/tex] is given by,

m = [tex]\frac{y_2-y_1}{x_2-x_1}[/tex]

Since, y-intercept and x-intercepts are equal,

Let the given line passes through (0, 3) and (-3, 0),

Slope of the line (m) = [tex]\frac{3-0}{0+3}[/tex] = 1

y-intercept of the line 'b' = 3

Equation of the line will be,

y = 1.x + 3

A group of 12 students is deciding whether to go to the science center of the zoo. Science center tickets are 3 for $36.75 and zoo tickets are 4 for $51.
How much will a group of 12 students save by choosing the science center?

Answers

Answer:

147

Step-by-step explanation:

If they go to the science center it will cost 3 people 36.75 cents so for 12 people it would be 36.75*4=147( I think let me know if I am wrong).

How many 8-song play lists contain no songs by One Direction? Suppose Janine decides to let her iPod select an 8-song play list at random.

Answers

Answer:

Step-by-step explanation:

Other Questions
If you had special powers, how would you have changed the Protestant Reformation. help as fast as you can!!!!!! Any living thing is called an organism,no matter if it is one-celled or many-celled. True or False?. What Law was passed by Henry VIII to make him the head of the Church What does the number 3 in 3p^5 represent? * Describe the transformation.* Dilation with a scale factor of 12* Dilation with a scale factor of 2* Dilation with a scale factor of 3* Dilation with a scale factor of 4* none of the above A test of a new car results in 300 miles on 10 gallons of gas. How far could you drive on 45 gallons of gas?Question 5 options:1350 miles67 miles670 miles135,000 miles Studio lighting is easy to carry around and make accessible. O True O False Which of the fractions is equivalent to 2/5 ?5/24/2512/154/10 Select the ordered pair that is a solution of the system of linear equations.y = 4x - 7y = -x +3o (1,2)o (2,1)o (1,1)o (2,2) 2. Tim and Stephanie are devastated when therecessive disease. Tim is shocked becausewould have gotten it from him. Is Tim cegenotypes must be in order for neither of them have the disease but have a son who does What is the set of interacting organs and tissues that fulfill one or more body functions ? what would you do if someone says that u were dating them and u were not the (blank???) was the Byzantine gold coin that was used from England to China Country G is a small industrialized nation in South America. The country's economy relies on oil exports and imports of durable goods. The government controls the oil industry and government spending in the economy accounts for $85 billion annually. Total exports are nearly $150 billion, while the household sector spends $65 billion on consumer goods and services. The country has a trade surplus of $40 billion.If the missing value is equal to $45 billion, what is Country G's GDP? can anyone help me on this? for algebra 2 please help I NEED HELP NOW PLZ! (ELA)look at the pic down below, can you give me the right order of those plz? How does eclipses occur Sunny made 2 pitchers of strawberry smoothies for the science club. If each person got One-fifth of a pitcher, how many science club members are there?2 area models. Both models are made up of 5 shaded parts.Which statements are true?Check all that apply.The correct division problem is One-fifth divided by 2.The correct division problem is 2 divided by one-fifth.The quotient isStartFraction 1 Over 10 EndFraction.The quotient is 10.There are 10 members in the science club.There are StartFraction 1 Over 10 EndFraction members in the science club. How do most primary producers make their own food? *